LSAT and Law School Admissions Forum

Get expert LSAT preparation and law school admissions advice from PowerScore Test Preparation.

User avatar
 Dave Killoran
PowerScore Staff
  • PowerScore Staff
  • Posts: 5852
  • Joined: Mar 25, 2011
|
#27098
Complete Question Explanation
(The complete setup for this game can be found here: lsat/viewtopic.php?f=325&t=6210)

The correct answer choice is (B)

Answer choice (A) is incorrect because M is not a member of either committee, a violation of the fourth rule.

Answer choice (C) is incorrect because according to the third rule if K is on a committee then J must be on that same committee, and the composition of the trails committee violates the rule.

Answer choice (D) is incorrect because the trails committee has only two members, a violation of the first rule.

Answer choice (E) is incorrect because F and K are both members of the planting committee, a violation of the second rule.

Thus, answer choice (B) is proven correct by process of elimination.

Get the most out of your LSAT Prep Plus subscription.

Analyze and track your performance with our Testing and Analytics Package.